Can every orthogonal matrix be written as a product of Givens rotations?












3














I'd like to know whether every orthogonal matrix



$$ A in mathcal{O}_n(mathbb{R})$$



can be written as a product of givens-rotations. I know that when we do QR-decomposition of matrix $A$ we get



$$ A = Q R $$



So my idea was to prove that $R$ must be the identity $I_n$, however I'm stuck at that. Can somebody give me a hint on how I could prove this?










share|cite|improve this question




















  • 3




    Orthogonal matrices could be reflections.
    – Doug M
    Nov 28 '18 at 18:01










  • www-old.math.gatech.edu/academic/courses/core/math2601/… page 38
    – Ryan Howe
    Nov 28 '18 at 18:06


















3














I'd like to know whether every orthogonal matrix



$$ A in mathcal{O}_n(mathbb{R})$$



can be written as a product of givens-rotations. I know that when we do QR-decomposition of matrix $A$ we get



$$ A = Q R $$



So my idea was to prove that $R$ must be the identity $I_n$, however I'm stuck at that. Can somebody give me a hint on how I could prove this?










share|cite|improve this question




















  • 3




    Orthogonal matrices could be reflections.
    – Doug M
    Nov 28 '18 at 18:01










  • www-old.math.gatech.edu/academic/courses/core/math2601/… page 38
    – Ryan Howe
    Nov 28 '18 at 18:06
















3












3








3







I'd like to know whether every orthogonal matrix



$$ A in mathcal{O}_n(mathbb{R})$$



can be written as a product of givens-rotations. I know that when we do QR-decomposition of matrix $A$ we get



$$ A = Q R $$



So my idea was to prove that $R$ must be the identity $I_n$, however I'm stuck at that. Can somebody give me a hint on how I could prove this?










share|cite|improve this question















I'd like to know whether every orthogonal matrix



$$ A in mathcal{O}_n(mathbb{R})$$



can be written as a product of givens-rotations. I know that when we do QR-decomposition of matrix $A$ we get



$$ A = Q R $$



So my idea was to prove that $R$ must be the identity $I_n$, however I'm stuck at that. Can somebody give me a hint on how I could prove this?







matrices matrix-decomposition orthogonal-matrices






share|cite|improve this question















share|cite|improve this question













share|cite|improve this question




share|cite|improve this question








edited Nov 28 '18 at 18:20









Jean-Claude Arbaut

14.7k63464




14.7k63464










asked Nov 28 '18 at 17:59









msrd0msrd0

1184




1184








  • 3




    Orthogonal matrices could be reflections.
    – Doug M
    Nov 28 '18 at 18:01










  • www-old.math.gatech.edu/academic/courses/core/math2601/… page 38
    – Ryan Howe
    Nov 28 '18 at 18:06
















  • 3




    Orthogonal matrices could be reflections.
    – Doug M
    Nov 28 '18 at 18:01










  • www-old.math.gatech.edu/academic/courses/core/math2601/… page 38
    – Ryan Howe
    Nov 28 '18 at 18:06










3




3




Orthogonal matrices could be reflections.
– Doug M
Nov 28 '18 at 18:01




Orthogonal matrices could be reflections.
– Doug M
Nov 28 '18 at 18:01












www-old.math.gatech.edu/academic/courses/core/math2601/… page 38
– Ryan Howe
Nov 28 '18 at 18:06






www-old.math.gatech.edu/academic/courses/core/math2601/… page 38
– Ryan Howe
Nov 28 '18 at 18:06












2 Answers
2






active

oldest

votes


















2














Givens rotations are... rotations, they preserve orientation ($det(M)=+1$), however $mathcal{O}_n(mathbb{R})$ has two components, one component is rotations ($det(M)=+1$), the other is reflections ($det(M)=-1$).



Example of $Minmathcal{O}_2(mathbb{R})$ that can not be written as a Givens rotation (its determinant is $-1$)
$$
M=left(begin{array}{cc}
-1 & 0 \
0 & 1
end{array}right)
$$

You can not find Givens rotation $G_theta=left(begin{array}{cc}
costheta & -sintheta \
sintheta & costheta
end{array}right)$
such that $M=G_theta$



If you want to have a Givens rotation decomposition you must restrict yourself to $SO_n(mathbb{R})$, the special orthogonal group, which is defined by $M^tM=I_d$ and $det{M}=+1$.






share|cite|improve this answer

















  • 1




    Thanks, I was trying to construct a matrix where it wouldn't work before attempting to prove, I must have missed that one
    – msrd0
    Nov 28 '18 at 22:13



















0














If you write the matrix $A=QR$ then there are two ways you can perform this.



$$ A underbrace{R_{1} R_{2} cdots R_{n}}_{hat{R}^{-1}} = hat{Q} tag{1} $$



or the following, which is what you want



$$ underbrace{Q_{n} Q_{n-1} cdots Q_{2}Q_{1}}_{{Q}^{*}}A = R tag{2} $$



In $1$ we see that Gram-Schmidt applies a sequence of elementary triangular matrices $R_{k}$ on the right of $A$. For $2$ we see a sequence of elementary unitary matrices $Q_{k}$ on the left of $A$. The matrices $Q_{k}$ can be givens matrices.



$$ underbrace{G_{n} G_{n-1} cdots G_{2}G_{1}}_{{G}^{*}}A = R tag{3} $$



All you need to prove is that $G_{k}$ is a unitary matrix. It is. There are notes provided here.






share|cite|improve this answer





















    Your Answer





    StackExchange.ifUsing("editor", function () {
    return StackExchange.using("mathjaxEditing", function () {
    StackExchange.MarkdownEditor.creationCallbacks.add(function (editor, postfix) {
    StackExchange.mathjaxEditing.prepareWmdForMathJax(editor, postfix, [["$", "$"], ["\\(","\\)"]]);
    });
    });
    }, "mathjax-editing");

    StackExchange.ready(function() {
    var channelOptions = {
    tags: "".split(" "),
    id: "69"
    };
    initTagRenderer("".split(" "), "".split(" "), channelOptions);

    StackExchange.using("externalEditor", function() {
    // Have to fire editor after snippets, if snippets enabled
    if (StackExchange.settings.snippets.snippetsEnabled) {
    StackExchange.using("snippets", function() {
    createEditor();
    });
    }
    else {
    createEditor();
    }
    });

    function createEditor() {
    StackExchange.prepareEditor({
    heartbeatType: 'answer',
    autoActivateHeartbeat: false,
    convertImagesToLinks: true,
    noModals: true,
    showLowRepImageUploadWarning: true,
    reputationToPostImages: 10,
    bindNavPrevention: true,
    postfix: "",
    imageUploader: {
    brandingHtml: "Powered by u003ca class="icon-imgur-white" href="https://imgur.com/"u003eu003c/au003e",
    contentPolicyHtml: "User contributions licensed under u003ca href="https://creativecommons.org/licenses/by-sa/3.0/"u003ecc by-sa 3.0 with attribution requiredu003c/au003e u003ca href="https://stackoverflow.com/legal/content-policy"u003e(content policy)u003c/au003e",
    allowUrls: true
    },
    noCode: true, onDemand: true,
    discardSelector: ".discard-answer"
    ,immediatelyShowMarkdownHelp:true
    });


    }
    });














    draft saved

    draft discarded


















    StackExchange.ready(
    function () {
    StackExchange.openid.initPostLogin('.new-post-login', 'https%3a%2f%2fmath.stackexchange.com%2fquestions%2f3017481%2fcan-every-orthogonal-matrix-be-written-as-a-product-of-givens-rotations%23new-answer', 'question_page');
    }
    );

    Post as a guest















    Required, but never shown

























    2 Answers
    2






    active

    oldest

    votes








    2 Answers
    2






    active

    oldest

    votes









    active

    oldest

    votes






    active

    oldest

    votes









    2














    Givens rotations are... rotations, they preserve orientation ($det(M)=+1$), however $mathcal{O}_n(mathbb{R})$ has two components, one component is rotations ($det(M)=+1$), the other is reflections ($det(M)=-1$).



    Example of $Minmathcal{O}_2(mathbb{R})$ that can not be written as a Givens rotation (its determinant is $-1$)
    $$
    M=left(begin{array}{cc}
    -1 & 0 \
    0 & 1
    end{array}right)
    $$

    You can not find Givens rotation $G_theta=left(begin{array}{cc}
    costheta & -sintheta \
    sintheta & costheta
    end{array}right)$
    such that $M=G_theta$



    If you want to have a Givens rotation decomposition you must restrict yourself to $SO_n(mathbb{R})$, the special orthogonal group, which is defined by $M^tM=I_d$ and $det{M}=+1$.






    share|cite|improve this answer

















    • 1




      Thanks, I was trying to construct a matrix where it wouldn't work before attempting to prove, I must have missed that one
      – msrd0
      Nov 28 '18 at 22:13
















    2














    Givens rotations are... rotations, they preserve orientation ($det(M)=+1$), however $mathcal{O}_n(mathbb{R})$ has two components, one component is rotations ($det(M)=+1$), the other is reflections ($det(M)=-1$).



    Example of $Minmathcal{O}_2(mathbb{R})$ that can not be written as a Givens rotation (its determinant is $-1$)
    $$
    M=left(begin{array}{cc}
    -1 & 0 \
    0 & 1
    end{array}right)
    $$

    You can not find Givens rotation $G_theta=left(begin{array}{cc}
    costheta & -sintheta \
    sintheta & costheta
    end{array}right)$
    such that $M=G_theta$



    If you want to have a Givens rotation decomposition you must restrict yourself to $SO_n(mathbb{R})$, the special orthogonal group, which is defined by $M^tM=I_d$ and $det{M}=+1$.






    share|cite|improve this answer

















    • 1




      Thanks, I was trying to construct a matrix where it wouldn't work before attempting to prove, I must have missed that one
      – msrd0
      Nov 28 '18 at 22:13














    2












    2








    2






    Givens rotations are... rotations, they preserve orientation ($det(M)=+1$), however $mathcal{O}_n(mathbb{R})$ has two components, one component is rotations ($det(M)=+1$), the other is reflections ($det(M)=-1$).



    Example of $Minmathcal{O}_2(mathbb{R})$ that can not be written as a Givens rotation (its determinant is $-1$)
    $$
    M=left(begin{array}{cc}
    -1 & 0 \
    0 & 1
    end{array}right)
    $$

    You can not find Givens rotation $G_theta=left(begin{array}{cc}
    costheta & -sintheta \
    sintheta & costheta
    end{array}right)$
    such that $M=G_theta$



    If you want to have a Givens rotation decomposition you must restrict yourself to $SO_n(mathbb{R})$, the special orthogonal group, which is defined by $M^tM=I_d$ and $det{M}=+1$.






    share|cite|improve this answer












    Givens rotations are... rotations, they preserve orientation ($det(M)=+1$), however $mathcal{O}_n(mathbb{R})$ has two components, one component is rotations ($det(M)=+1$), the other is reflections ($det(M)=-1$).



    Example of $Minmathcal{O}_2(mathbb{R})$ that can not be written as a Givens rotation (its determinant is $-1$)
    $$
    M=left(begin{array}{cc}
    -1 & 0 \
    0 & 1
    end{array}right)
    $$

    You can not find Givens rotation $G_theta=left(begin{array}{cc}
    costheta & -sintheta \
    sintheta & costheta
    end{array}right)$
    such that $M=G_theta$



    If you want to have a Givens rotation decomposition you must restrict yourself to $SO_n(mathbb{R})$, the special orthogonal group, which is defined by $M^tM=I_d$ and $det{M}=+1$.







    share|cite|improve this answer












    share|cite|improve this answer



    share|cite|improve this answer










    answered Nov 28 '18 at 18:14









    Picaud VincentPicaud Vincent

    1,29539




    1,29539








    • 1




      Thanks, I was trying to construct a matrix where it wouldn't work before attempting to prove, I must have missed that one
      – msrd0
      Nov 28 '18 at 22:13














    • 1




      Thanks, I was trying to construct a matrix where it wouldn't work before attempting to prove, I must have missed that one
      – msrd0
      Nov 28 '18 at 22:13








    1




    1




    Thanks, I was trying to construct a matrix where it wouldn't work before attempting to prove, I must have missed that one
    – msrd0
    Nov 28 '18 at 22:13




    Thanks, I was trying to construct a matrix where it wouldn't work before attempting to prove, I must have missed that one
    – msrd0
    Nov 28 '18 at 22:13











    0














    If you write the matrix $A=QR$ then there are two ways you can perform this.



    $$ A underbrace{R_{1} R_{2} cdots R_{n}}_{hat{R}^{-1}} = hat{Q} tag{1} $$



    or the following, which is what you want



    $$ underbrace{Q_{n} Q_{n-1} cdots Q_{2}Q_{1}}_{{Q}^{*}}A = R tag{2} $$



    In $1$ we see that Gram-Schmidt applies a sequence of elementary triangular matrices $R_{k}$ on the right of $A$. For $2$ we see a sequence of elementary unitary matrices $Q_{k}$ on the left of $A$. The matrices $Q_{k}$ can be givens matrices.



    $$ underbrace{G_{n} G_{n-1} cdots G_{2}G_{1}}_{{G}^{*}}A = R tag{3} $$



    All you need to prove is that $G_{k}$ is a unitary matrix. It is. There are notes provided here.






    share|cite|improve this answer


























      0














      If you write the matrix $A=QR$ then there are two ways you can perform this.



      $$ A underbrace{R_{1} R_{2} cdots R_{n}}_{hat{R}^{-1}} = hat{Q} tag{1} $$



      or the following, which is what you want



      $$ underbrace{Q_{n} Q_{n-1} cdots Q_{2}Q_{1}}_{{Q}^{*}}A = R tag{2} $$



      In $1$ we see that Gram-Schmidt applies a sequence of elementary triangular matrices $R_{k}$ on the right of $A$. For $2$ we see a sequence of elementary unitary matrices $Q_{k}$ on the left of $A$. The matrices $Q_{k}$ can be givens matrices.



      $$ underbrace{G_{n} G_{n-1} cdots G_{2}G_{1}}_{{G}^{*}}A = R tag{3} $$



      All you need to prove is that $G_{k}$ is a unitary matrix. It is. There are notes provided here.






      share|cite|improve this answer
























        0












        0








        0






        If you write the matrix $A=QR$ then there are two ways you can perform this.



        $$ A underbrace{R_{1} R_{2} cdots R_{n}}_{hat{R}^{-1}} = hat{Q} tag{1} $$



        or the following, which is what you want



        $$ underbrace{Q_{n} Q_{n-1} cdots Q_{2}Q_{1}}_{{Q}^{*}}A = R tag{2} $$



        In $1$ we see that Gram-Schmidt applies a sequence of elementary triangular matrices $R_{k}$ on the right of $A$. For $2$ we see a sequence of elementary unitary matrices $Q_{k}$ on the left of $A$. The matrices $Q_{k}$ can be givens matrices.



        $$ underbrace{G_{n} G_{n-1} cdots G_{2}G_{1}}_{{G}^{*}}A = R tag{3} $$



        All you need to prove is that $G_{k}$ is a unitary matrix. It is. There are notes provided here.






        share|cite|improve this answer












        If you write the matrix $A=QR$ then there are two ways you can perform this.



        $$ A underbrace{R_{1} R_{2} cdots R_{n}}_{hat{R}^{-1}} = hat{Q} tag{1} $$



        or the following, which is what you want



        $$ underbrace{Q_{n} Q_{n-1} cdots Q_{2}Q_{1}}_{{Q}^{*}}A = R tag{2} $$



        In $1$ we see that Gram-Schmidt applies a sequence of elementary triangular matrices $R_{k}$ on the right of $A$. For $2$ we see a sequence of elementary unitary matrices $Q_{k}$ on the left of $A$. The matrices $Q_{k}$ can be givens matrices.



        $$ underbrace{G_{n} G_{n-1} cdots G_{2}G_{1}}_{{G}^{*}}A = R tag{3} $$



        All you need to prove is that $G_{k}$ is a unitary matrix. It is. There are notes provided here.







        share|cite|improve this answer












        share|cite|improve this answer



        share|cite|improve this answer










        answered Nov 28 '18 at 18:15









        Ryan HoweRyan Howe

        2,41911323




        2,41911323






























            draft saved

            draft discarded




















































            Thanks for contributing an answer to Mathematics Stack Exchange!


            • Please be sure to answer the question. Provide details and share your research!

            But avoid



            • Asking for help, clarification, or responding to other answers.

            • Making statements based on opinion; back them up with references or personal experience.


            Use MathJax to format equations. MathJax reference.


            To learn more, see our tips on writing great answers.





            Some of your past answers have not been well-received, and you're in danger of being blocked from answering.


            Please pay close attention to the following guidance:


            • Please be sure to answer the question. Provide details and share your research!

            But avoid



            • Asking for help, clarification, or responding to other answers.

            • Making statements based on opinion; back them up with references or personal experience.


            To learn more, see our tips on writing great answers.




            draft saved


            draft discarded














            StackExchange.ready(
            function () {
            StackExchange.openid.initPostLogin('.new-post-login', 'https%3a%2f%2fmath.stackexchange.com%2fquestions%2f3017481%2fcan-every-orthogonal-matrix-be-written-as-a-product-of-givens-rotations%23new-answer', 'question_page');
            }
            );

            Post as a guest















            Required, but never shown





















































            Required, but never shown














            Required, but never shown












            Required, but never shown







            Required, but never shown

































            Required, but never shown














            Required, but never shown












            Required, but never shown







            Required, but never shown







            Popular posts from this blog

            Quarter-circle Tiles

            build a pushdown automaton that recognizes the reverse language of a given pushdown automaton?

            Mont Emei